www.vorhilfe.de
Vorhilfe

Kostenlose Kommunikationsplattform für gegenseitige Hilfestellungen.
Hallo Gast!einloggen | registrieren ]
Startseite · Forum · Wissen · Kurse · Mitglieder · Team · Impressum
Forenbaum
^ Forenbaum
Status Vorhilfe
  Status Geisteswiss.
    Status Erdkunde
    Status Geschichte
    Status Jura
    Status Musik/Kunst
    Status Pädagogik
    Status Philosophie
    Status Politik/Wirtschaft
    Status Psychologie
    Status Religion
    Status Sozialwissenschaften
  Status Informatik
    Status Schule
    Status Hochschule
    Status Info-Training
    Status Wettbewerbe
    Status Praxis
    Status Internes IR
  Status Ingenieurwiss.
    Status Bauingenieurwesen
    Status Elektrotechnik
    Status Maschinenbau
    Status Materialwissenschaft
    Status Regelungstechnik
    Status Signaltheorie
    Status Sonstiges
    Status Technik
  Status Mathe
    Status Schulmathe
    Status Hochschulmathe
    Status Mathe-Vorkurse
    Status Mathe-Software
  Status Naturwiss.
    Status Astronomie
    Status Biologie
    Status Chemie
    Status Geowissenschaften
    Status Medizin
    Status Physik
    Status Sport
  Status Sonstiges / Diverses
  Status Sprachen
    Status Deutsch
    Status Englisch
    Status Französisch
    Status Griechisch
    Status Latein
    Status Russisch
    Status Spanisch
    Status Vorkurse
    Status Sonstiges (Sprachen)
  Status Neuerdings
  Status Internes VH
    Status Café VH
    Status Verbesserungen
    Status Benutzerbetreuung
    Status Plenum
    Status Datenbank-Forum
    Status Test-Forum
    Status Fragwürdige Inhalte
    Status VH e.V.

Gezeigt werden alle Foren bis zur Tiefe 2

Navigation
 Startseite...
 Neuerdings beta neu
 Forum...
 vorwissen...
 vorkurse...
 Werkzeuge...
 Nachhilfevermittlung beta...
 Online-Spiele beta
 Suchen
 Verein...
 Impressum
Das Projekt
Server und Internetanbindung werden durch Spenden finanziert.
Organisiert wird das Projekt von unserem Koordinatorenteam.
Hunderte Mitglieder helfen ehrenamtlich in unseren moderierten Foren.
Anbieter der Seite ist der gemeinnützige Verein "Vorhilfe.de e.V.".
Partnerseiten
Dt. Schulen im Ausland: Mathe-Seiten:

Open Source FunktionenplotterFunkyPlot: Kostenloser und quelloffener Funktionenplotter für Linux und andere Betriebssysteme
Forum "Uni-Analysis-Komplexe Zahlen" - Argument
Argument < Komplexe Zahlen < Analysis < Hochschule < Mathe < Vorhilfe
Ansicht: [ geschachtelt ] | ^ Forum "Uni-Analysis-Komplexe Zahlen"  | ^^ Alle Foren  | ^ Forenbaum  | Materialien

Argument: Aufgabe 1
Status: (Frage) beantwortet Status 
Datum: 16:37 Di 04.01.2011
Autor: Random

Aufgabe
Bestimmen Sie Real-, Imaginäranteil, Betrag und Argument von der Komplexen Zahl:

z=-2+2*i


Guten Tag Matheraum =)

Also Rez=-2 Imz=2.

Der Betrag: [mm] \wurzel{(-2)^2+2^2}=2*\wurzel{2} [/mm]

Puh das wäre geschafft xD...

Beim Argument habe ich Verständnisprobleme.

Ich weiss, dass [mm] \tan(\alpha)=\bruch{Imz}{Rez} [/mm]

Und somit ist der Winkel [mm] \alpha=\tan^{-1}\bruch{Imz}{Rez}=-0,78 [/mm]

Irgendwas läuft schief.... Ich verstehe auch nicht warum manchmal [mm] \pi [/mm] dazugerechnet wird also in welchen Fällen das passiert.

Vielen Dank schon mal im Voraus,

Ilya




        
Bezug
Argument: Tipps
Status: (Antwort) fertig Status 
Datum: 16:58 Di 04.01.2011
Autor: Infinit

Hallo Ilya,
der Arcustangens ist zwar eindeutig, aber durch die Bruchbildung von Imaginär- und Realteil entsteht ein Bruch, der a) entweder positiv oder b) negativ ist. Dem Ergbnis selbst kannst du leider nicht ansehen, ob es deswegen positiv ist, weil Real- und Imaginärteil beide positiv waren oder ob dies deswegen der Fall ist, weil beide Anteile der komplexen Zahl negativ waren. Entsprechend ist es bei einem negativen Ergebnis, denn dieses kann durch eine komplexe Zahl zustande kommen, die entweder im zweiten oder im vierten Quadranten liegt. Insofern muss man nach der Bestimmung des Arcustangens immer noch mal kontrollieren, ob man nicht um 180 Grad daneben liegt.    
In Deinem Beispiel mit
[mm] \varphi = \arctan (\bruch{-2}{2}) = - 1 [/mm]
bekommst Du - 45 Grad heraus, hier wäre der Realteil positiv und der Imaginärteil negativ. Das stimmt aber nicht mit Deiner gegebenen komplexen Zahl überein, die im 2. Quadranten liegt. Hier musst Du also nochmal 180 Grad dazuaddieren, der richtige Winkel ist demzufolge 135 Grad.
Viele Grüße,
Infinit


Bezug
                
Bezug
Argument: Frage (beantwortet)
Status: (Frage) beantwortet Status 
Datum: 17:05 Di 04.01.2011
Autor: Random

Danke sehr dann habe ich noch zwei Fragen =):

1) Wieso ist [mm] arctan(\bruch{-2}{2})=arctan(-1)=-45° [/mm] ?

Also wie kommt man da auf -45°? Es kommt ja -0,78 oder so raus...

2) Also wenn ich die komplexe Zahl anschaue und sehe aha sie liegt entweder im 2. oder im 4. Quadranten kann ich den Winkel immer um 180° also um [mm] \pi [/mm] erhöhen?

In meinem Beispiel gehe ich -2 nach links und 2 nach oben im Koordinatensystem und sehe, dass die Zahl im 2. Quadranten liegt und addiere dann [mm] \pi [/mm] auf die -45° (von den ich noch nicht weiss wie die bestimmt wurden)?

Vielen Dank nochmal im Voraus,

Ilya

Bezug
                        
Bezug
Argument: Bogenmaß
Status: (Antwort) fertig Status 
Datum: 17:16 Di 04.01.2011
Autor: Infinit

Hallo ilya,
augenscheinlich hast Du Deinen Rechner im Bogenmaß laufen, dann kommt dieser Wert raus. In Grad umgerechnet sind es - 45 Grad.
Natürlich addierst Du nicht immer 180 Grad dazu, falls die Werte schon im richtigen Quadranten liegen, macht dies natürlich keinen Sinn.
Ein einfaches Beispiel:
Betrachte die komplexe Zahl
[mm] z = 2 - 2i [/mm]
Diese Zahl liegt bereits im vierten Quadranten, Dein Ergebnis von -45 Grad stimmt also in diesem Fall.
Viele Grüße,
Infinit  


Bezug
                                
Bezug
Argument: Frage (beantwortet)
Status: (Frage) beantwortet Status 
Datum: 17:31 Di 04.01.2011
Autor: Random

Danke sehr!

Ja hab das mit Bogenmaß im Rechner komplet vergessen xD.

zu 2) Ich habe nicht ganz verstanden wie ich herausfinde, ob mein Ergebnis im richtigen Quadranten liegt oder nicht. In deinem Tipps-Beitrag hast du es zwar erklärt, aber irgendwie wird es mir nicht ganz klar.

Nach welchen Kriterien muss ich schauen? Muss ich gucken, welche Vorzeichen die beiden Anteile der komplexen Zahl haben? Wenn ja was sagen mir diese?

Also bei 2-2i bin ich, wie du sagtest, im richtigen Quadranten und muss nicht erhöhen

und bei -2+2i im falschen und muss erhöhen, aber wieso? xD

Bei 2-2i: [mm] \bruch{-2}{2} [/mm]
Bei -2+2i: [mm] \bruch{2}{-2} [/mm]

Kann ich hier schon sehen, ob ich im richtigen Quadranten bin?

Vielen Dank im Voraus,

Ilya

Bezug
                                        
Bezug
Argument: Erkennbar
Status: (Antwort) fertig Status 
Datum: 17:41 Di 04.01.2011
Autor: Infinit

Hallo ilya,
an dem noch nicht ausgerechneten Bruch kannst Du erkennen, wo die komplexe Zahl liegt, denn Du weisst, dass im Zähler der Imaginärteil und im Nenner der Realteil steht. Beide Beispiele ergeben jedoch den Wert -1 und wenn du nur diesen Wert betrachtest, dann ist eben die Mehrdeutigkeit wieder da und Du musst mit der ursprünglichen Ausgangszahl Dein Ergebnis vergleichen.
Viele Grüße,
Infinit



Bezug
                                                
Bezug
Argument: Frage (beantwortet)
Status: (Frage) beantwortet Status 
Datum: 17:49 Di 04.01.2011
Autor: Random

Okay also wenn ich die Zahl -2+2i habe weiss ich, dass diese im 2.Quadranten liegt, jedoch liegt die Lösung arctan(-1) im 4. Quadranten. Also erhöhe ich um [mm] \pi [/mm] um in den 2. Quadranten zu kommen.

Bei der Zahl 2-2i weiss ich, dass diese im 4. Quadranten liegt und die Lösung -45° ebenfalls also muss ich nichts erhöhen.

Habe ich das richtig verstanden?

Ilya

Bezug
                                                        
Bezug
Argument: Antwort
Status: (Antwort) fertig Status 
Datum: 18:00 Di 04.01.2011
Autor: MathePower

Hallo Random,

> Okay also wenn ich die Zahl -2+2i habe weiss ich, dass
> diese im 2.Quadranten liegt, jedoch liegt die Lösung
> arctan(-1) im 4. Quadranten. Also erhöhe ich um [mm]\pi[/mm] um in
> den 2. Quadranten zu kommen.


>
> Bei der Zahl 2-2i weiss ich, dass diese im 4. Quadranten
> liegt und die Lösung -45° ebenfalls also muss ich nichts
> erhöhen.
>
> Habe ich das richtig verstanden?
>


Ja, das hast Du.


> Ilya


Gruss
MathePower

Bezug
                                                        
Bezug
Argument: Okay
Status: (Mitteilung) Reaktion unnötig Status 
Datum: 18:00 Di 04.01.2011
Autor: Infinit

Hallo Ilya,
so ist es. Immer gegenchecken, um sicherzugehen.
Viele Grüße,
Infinit


Bezug
                                                                
Bezug
Argument: Danke
Status: (Mitteilung) Reaktion unnötig Status 
Datum: 18:17 Di 04.01.2011
Autor: Random

Vieln Dank nochmal Infinit!!!

Bezug
Ansicht: [ geschachtelt ] | ^ Forum "Uni-Analysis-Komplexe Zahlen"  | ^^ Alle Foren  | ^ Forenbaum  | Materialien


^ Seitenanfang ^
www.vorhilfe.de